On Friday,2,364 cars parked in the garage at the mall. On Sunday 2,455 more cars parked In the garage than on Friday. How many cars parked in the garage during the two days?

Answers

Answer 1

Answer:

7183 cars

Step-by-step explanation:

On Friday, 2364 cars parked in the garage.

On Sunday, 2,455 more cars parked In the garage than on Friday.

This means that the number of cars parked on Sunday is 2455 plus the number of cars parked on Friday, that is:

2455 + 2364 = 4819 cars

Therefore, the number of cars parked in the garage in the two days is:

4819 + 2364 = 7183 cars

7183 cars were parked during the two days.


Related Questions

solve the equation 18y - 17 = 7 for y

Answers

Answer:

y = 1.3

Step-by-step explanation:

18y - 17 = 7

18y = 7 + 17

18y = 24

y = 24/18

y = 1.3

hope it helps!

Answer:

y=4/3

Step-by-step explanation:

To solve for y, we need to isolate it.

18y-17=7

Let's start by adding 17 to both sides.

18y=24

Next, we can divide both sides by 18.

y=24/18

Simplify.

y=4/3

Georgianna wants to use the linear model associated with the data in the table to make a prediction.

A 2-column table with 5 rows. The first column is labeled time (minutes) with entries 0, 5, 10, 15, 30. The second column is labeled distance (miles) with entries 0, 4, 9, 13, 18.
Which range of time values describes the entire interval over which she would be interpolating?

0 to 5 minutes
0 to 18 minutes
0 to 30 minutes
18 to 30 minutes

Answers

Answer:

0 to 30 minutes on edg.

Step-by-step explanation:

Answer:

c on e 2020

Step-by-step explanation:

If 3x and 71/x are two prime numbers V x equivalent to R, then number of x so that 3x + 71/x = 10 is/ are

Answers

Answer:

x = 5/3

Step-by-step explanation:

Guven two prime numbers to be 31/x and 71/x, if their sum is 10 as given;

3x + 71/x = 10 then to find the value of x, the following steps must be taken;

Step 1

Find the LCM of the given equation;

3x + 71/x = 10

[tex]\frac{3x^{2}+71 }{x} =10\\[/tex]

Step 2:

Cross multiplying;

[tex]3x^{2} +71=10x\\3x^{2} -10x+71 =0\\[/tex]

Using the general formula to get the value of x;

x = -b±√b²-4ac/2a

a=3, b=-10, c=71

= 10±√(-10)²-4(3)(71)/2(3)

= 10±√100-852/6

= 10±√-752/6

= 10±27.4i/6

= 10+27.4i/6 or 10-27.4i/6

x = 5/3+27.4i/6 or 5/3-27.4i/6

Since the values of x are real values then, our answer will be the real part of the complex number gotten.

x = 5/3

Find an explicit rule for the nth term of a geometric sequence where the second and fifth terms are -36 and 2304, respectively.

Answer Choices:

A) an = 9 • 4n

B) an = 9 • (-4)n + 1

C) an = 9 • 4n - 1

D) an = 9 • (-4)n - 1

Answers

It’s probably B but I’m sure yet

Which of the following options have the same value as %25, percent of 64? Choose 2 answers:
A 25⋅64
B 25/100⋅64
C 0.25⋅64
D 64÷ 25/100
E 25÷ 64/100

Answers

Answer:

B and C

Step-by-step explanation:

25% of 64 is 16. So find the two answers that are equivalent to 16.

A: 25*64 = 1600 [tex]\neq[/tex] 16

B: [tex]\frac{25}{100}[/tex]*64 = 16

C: 0.25 * 64 = 16

D: 64/[tex]\frac{25}{100}[/tex] =256 [tex]\neq[/tex] 16

E: 25/[tex]\frac{64}{100}[/tex] = 39.06 [tex]\neq[/tex] 16

Answer:

its b and c. had the same problem.

EMERGENCY!!!! EXTRA POINTS! I HAVE POSTED THIS QUESTION 5 TIMES AND HAD TO REPORT ALL RESPONSES! I ACTUALLY NEED HELP!
The amusement park in Riverdale charges a $10 entrance fee plus $1 per ride. The amusement park in Esterdamb charges a $5 entrance fee and $2 per ride. For how many rides will the cost be the same for both amusement parks? x= # of rides
Number of rides_____?
Cost_____?

Answers

Answer:

the price will be the same for either amusement park

Step-by-step explanation:

10+1(x)=?

5+2(x)=?

ex; x=5

10+1(5)=                                   5+2(5)=

10+5=                                      5+10=

15                                            15

Answer:

5 rides, cost of $15

Step-by-step explanation:

With the information given we have two expressions:

10 + 1x and 5 + 2x

Using these expressions we can create this equation:

10 + 1x = 5 + 2x

Knowing some information we can can simply analyze some information:

The closest number each of these equations could equal is 15,

1(5) is 5 + 10 = 15, and 2(5) is 10 + 5 = 15

15 = 15 this is a true statement, so we know this is answer

Hope this helps! Have a good day!

help plz! (a + b - c)(a + b + c) multiplying polynomials

Answers

Answer:

a^2 + 2ab + b^2 - c^2

Step-by-step explanation:

When you multiply all the terms together you get a^2 + ab + ac + ab + b^2 + bc - ac - bc - c^2. Then you can just combine like terms and simplify it to a^2 + 2ab + b^2 - c^2. Hope this helps :)

Given that x+1 is a factor of 2x^3-3x^2-8x-3 , solve 2x^3-3x^2-8x-3=0. This means find all the roots for the function.

Answers

Answer:

x=-1, -1/2, 3

Step-by-step explanation:

Find the total volume of the rectangular prisms.
A) 88 units3
B) 96 units3
C) 104 units3
D) 114 units3

Answers

Answer:

C) 104 units^3

Step-by-step explanation:

The formula for volume in a rectangular prism is V=lwh

There are three total rectangular prisms, so take the volume of each individual prism and add them together to get the total volume.

smallest: V=(2)(2)(4) = 16

Medium: (2)(2)(6) = 24

Largest: (2)(2)(16) = 64

Add it together: 16+24+64= 104

Your answer is 104 units^3

:)

2. 2x - y = 6,
- x + y = -1

Answers

Answer:

x = 5

y = 4

Step-by-step explanation:

just substitute the value of one variable into the other

-x +y = -1

or y = -1 +x   <-------- (i just added x to the right side)

now u know what y is equal to, just substitute that value into the first equation

2x - (-1+x) = 6     <-------- instead of y, i put the value of y from the 2 equation

NOW SOLVE FOR X!

2x +1 -x = 6

x + 1 = 6

x = 5

now solve for y since you have the value of x

y = -1 + x

or y = -1 + 5

y = 4

Answer:

y=3.166     x=4.166

Step-by-step explanation:

Factor the expression 2x4y – 18x2y3 completely.

A. 2x2y(x2 – 9y2)

B. 2x2y(x – 3y)(x + 3y)

C. 2x2y(x – 3y)2

D. 2x2y(x – 9y)(x + 2y)

Answers

Answer:

B

Step-by-step explanation:

[tex]2x^{4}y - 18x^{2}y^{3}[/tex]

Both sides have [tex]x^{2}[/tex]

[tex]x^{2} (2x^{2}y - 18y^{3})[/tex]

Both sides have y

[tex]yx^{2} (2x^{2} - 18y^{2})[/tex]

Both sides have 2

[tex]2yx^{2} (x^{2} - 9y^{2})[/tex]

Rule : [tex]x^{2} - y^{2} = (x+y)(x -y)[/tex]

[tex]x^{2} = x.x\\9y^{2} = 3y. 3y[/tex]

[tex](x^{2} - 9y^{2}) = (x +3y) (x-3y)[/tex]

Final:

[tex]\red{ 2 {x}^{2} y(x - 3y)(x + 3y)}[/tex]

Hope this helps ^-^

The factored expression is 2x²y(x + 3y)(x- 3y), and option (B) is correct.

What is Factorization?

The act of expressing a number or other mathematical object as the sum of numerous factors is known as factorization.

The given expression is 2x⁴y-18x²y³.

Factor the expression as follows:

2x⁴y-18x²y³

= 2x²y(x² - 9y²)

= 2x²y(x² - (3y)²)

= 2x²y(x + 3y)(x- 3y)

Hence, the factored expression is 2x²y(x + 3y)(x- 3y), and option (B) is correct.

Learn more about factorization:

https://brainly.com/question/24182713

#SPJ5

Find the slope between:
(4, -1)
(-2,4)

Answers

Answer:

-5

----

6

Step-by-step explanation:

y2-y1

-------

x2-x1

-1-4 -5

------ = ---

4--2 6

~Anna

What is the area of the triangle (I need this today pls help)

Answers

Answer: 24 units^2

(12*4)/(2)=24

Step-by-step explanation:

Answer:

Step-by-step explanation:

4*12/0.5=96

Derek is playing a prank on Grace and hid her pencil inside a tissue box that has a width of 4 inches, a length of 8 inches, and a height of 4 inches. The pencil fits perfectly from the upper left corner to the bottom right corner of the box. What is the length of Grace’s pencil to the nearest tenth of an inch?

Answers

Answer:

9.8 Inch

Step-by-step explanation:

The distance from the upper left corner to the bottom right corner of the box is the longest diagonal of the box.

For a rectangular prism:

Length of the Longest diagonal [tex]=\sqrt{ l^2+h^2+w^2}[/tex]

Therefore:

Length of the pencil[tex]=\sqrt{ 8^2+4^2+4^2}[/tex]

[tex]=\sqrt{96}\\=9.8$ inch(correct to the nearest tenth of an inch)[/tex]

Answer: length of pencil is 9.8 inches.

Step-by-step explanation:

The pencil fits perfectly from the upper left corner to the bottom right corner of the box. This is the diagonal of the box(not the diagonal of a face). To determine the length of the pencil, we would determine the length of the diagonal of the box. The formula formula is

d = √(l² + w² + h²)

Where

d = diagonal = length of pencil

w = width = 4 inches

h = height = 4 inches

l = length = 8 inches

Therefore,

d = √(4² + 4² + 8²) = √96

d = 9.8 inches

when nathan was 2 1/2 years old, he was 34 inches tall. the doctor told his mother that he will be two times and 4 inches taller than that when he is an adult. which of the following equation will give h, nathans height, as an adult

Answers

Answer 5 ft 4 in

Step-by-step explanation:

2 1/2 + 2 1/2=5 + 4 inches

You start a chain email and send it to 3 friends. The process continues and each of your friends forward the email to 3 other people. What is an expression for the nth term an?

Answers

Answer:

[tex]a_{n}=3^{n}[/tex]

Step-by-step explanation:

We can start solving this problem by making a list of the number of chain emails that are sent on each round:

Round one:

[tex]a_{1}=3[/tex]

which represents the three friends you send the mail to.

Round two:

[tex]a_{2}=3*3=9[/tex]

since each of your friends sends the mail to another three friends, then we multiply the original 3 friends by 3 to get a total of 9.

Round three:

[tex]a_{3}=3*3*3=9*3=27[/tex]

on the next round we multiply the previous 9 persons by 3 mails each giving us a total of 27 persons receiving the mail. A patter starts showing here. Notice that for each round we need to take the previous round number an multiply it by 3. When we have a number being multiplied by itself n times, we rewrite it as a power, so the expression for the nth term will be:

[tex]a_{n}=3^{n}[/tex]

which is our final answer.

What is the distance between points A(13, 2) and B(7, 10)

Answers

Answer:

distance = sqrt((x2-x1)^2 + (y2-y1))

distance = sqrt((7-13)^2 + (10-2)^2)

distance = sqrt( -6^2 + 8^2)

distance = sqrt(36+64)

distance = 10

Step-by-step explanation:

Answer:

10 units

Step-by-step explanation:

d = [tex]\sqrt{(x2 - x1)^2 + (y2 - y1)^2}[/tex]

d = [tex]\sqrt{(13 - 7)^2 + (2 - 10)^2}[/tex]

d = [tex]\sqrt{(6)^2 + (-8)^2}[/tex]

d = [tex]\sqrt{36 + 64}[/tex]

d = [tex]\sqrt{100}[/tex]

d = 10 units

Solve 8.2ws 29.52. Which of the following must be true abo
OWS36
The arrow points left
The arrow points right
There is an open circle at 3.6.
There is a closed circle at 3.6.

Answers

Answer:

A B E

Step-by-step explanation:

A- w less-than-or-equal-to 3.6

B- The arrow points left.

E- There is a closed circle at 3.6.

Answer:

A B E

Step-by-step explanation:

pls helllllllllllllp

Answers

Answer:

40

Step-by-step explanation:

24 divided by = 8

8 times 5 = 40

can i have brainliest please

Answer:

40

Step-by-step explanation:

there is a 5 to 3 ratio for ice cream cones to brownies

ice cream: brownies

5:3

x:24

three times eight is 24

five times eight is 40

find angle C! please and thankyou

Answers

Answer: it's around 40

Step-by-step explanation:

Sumara Kato's savings account has a balance of $1663. After 19 years, how much interest
will she have earned if her rate is 5.5% compounded annually?
(1) $3159.70 (2) $2936.27 (3) $2942.27
(4) $2925.27

Answers

Answer: $2936.27

Step-by-step explanation:

Present value = $1663

Rate = 5.5%

Time(n) = 19years

Future value= PV(1+r)^n

= 1663(1+5.5%)^19

= 1663(1.055)^19

=1663 × 2.766

=4599.86

Interest = Future value - Present value

= $4599 - $1663

= 2936

the graphs of f(x)=-2x and g(x)=(1/2)^x are shown. what are the solutions to thr equation -2x=(1/2)^x? select each correct answer.
-2
-1
2
4

Answers

Answer:

-2

Step-by-step explanation:

hello :

f(x)=-2x and g(x)=(1/2)^x

f(-2)=-2(-2) =4

g(-2)=(1/2)^-2 = 1/2^-2 = 2²=4

Carlos bought 80 feet of wire for ​$17.60. He needs 30 more feet. If the unit price is the​ same, how much will he pay for the extra 30 feet of wire​?

Answers

Answer:

Carlos bought 80 feet of wire for ​$17.60.

Then the price for 1 feet of wire is 17.6/80 = 0.22

=> 30 feet of wire would cost: 30 x 0.22 = 6.6

Hope this helps!

:)

Which statement is true about a line that contains the point ሺ2, 7ሻ and has a slope of 0.5?

A The line will also contain the point ሺ4, 6ሻ.
B The line will have a y-intercept of 6.
C The line will contain points whose y-coordinate will decrease in value as you move to the right
D none of the above

Answers

Answer:

I think it is C) The line will contain points whose y-coordinate will decrease in value as you move to the right

Step-by-step explanation:

HOPE THIS HELPS

If you are given a paper which has lines which are the length of a needle apart, and then you repeated drop that needle onto the paper, the probability that the needle with cut the line is:

Answers

Answer:

The correct option is;

[tex]\frac{1}{\pi }[/tex]

Step-by-step explanation:

Here we have that

[tex]Probability = \frac{Number \, of \, required\, outcomes}{Number \, of \, possible\ outcomes} = \frac{Dimension \, of \, the \, line}{Size \, of \, the \ needle} = \frac{l \times D}{\pi \times D \times l } = \frac{1}{\pi }[/tex]

Therefore, the probability that the needle will cut the line = 1/π.

Mr. Stewart used a gallon of vinegar to make 15 different color egg dipping containers. He used the same amount of vinegar in each container. How much vinegar in gallons was used in each egg dye container?

Answers

Answer:

0.067 gallons ( to 2 decimal places)

Step-by-step explanation:

First, note that since the same amount of vinegar was used in each container, it means that the 1 gallon of vinegar was divided equally to the 15 containers, hence the amount of vinegar used in each dye container is calculated as follows:

total volume of vinegar = 1

number of egg dye containers = 15

if equal volume of vinegar was used in each container:

volume of vinegar in each egg dye container = 1 ÷ 15 = 0.067 gallons ( to 2 decimal places)

I need help with these plzzz

Answers

Answer:

1. C 2. B

Step-by-step explanation:

1. find the total circumference, d times pi, then put 42 over 360 and multiply them to get that section

2. The actual value of x is 33.2

So I plugged in that value for all of them

The Answer is 1.c 2.b

6 radians is the same as____
Round to the nearest hundredth of a degree.

Answers

Answer:

343.77

Step-by-step explanation:

1 radian is about 57.2958 degrees

so 57 times 6 is about 343.77

343.77 is your answer

A savings account balance is compounded continuously.If the interest rate is 3.1% per year and the current balance is 1077.00 in how many years will the balance reach 1486.73?

Answers

Answer:13.8 is the best answer i could get

Step-by-step explanation:1,077 multiply that × 13.8 =14,862

Which represents the solution(s) of the system of equations, y = -x2 + 6x + 16 and y = - 4x + 37? Determine the solution set
algebraically
O (325)
(-3, 49)
(3,25) and (79)
(-3, 49) and (-7, 65)

Answers

Answer:

The answer is c

Step-by-step explanation:

(7,9) (3,25)

The solution sets of the given system of equations are (3,25) and (7,9).

Given equations are :

[tex]y=-x^{2} +6x+16[/tex]........(1)

[tex]y=-4x+37[/tex]...........(2)

What is an equation?

An equation is a statement that equates to two expressions.

So, for the solutions of the  given system of equations

[tex]-x^{2} +6x+16=-4x+37[/tex]

[tex]x^{2} -10x+21=0[/tex]

[tex]x^{2} -3x-7x+21=0\\x(x-3)-7(x-3)=0[/tex]

[tex](x-3)(x-7)=0[/tex]

So, [tex]x=3[/tex]

[tex]x=7[/tex]

Corresponding y value for x=3

[tex]y=-4*3+37\\y=25[/tex]

Corresponding y value for x=7

[tex]y=-4*7+37[/tex]

[tex]y=9[/tex]

Hence, the solution sets of the given system of equations are (3,25) and (7,9).

To get more about equations visit:

https://brainly.com/question/14323743

Other Questions
The area of Miguels rectangular garden is 450 square feet. The garden is 9 feet wide. How many feet of fencing will Miguel need to buy to enclose the garden on all four sides? In MNO, the measure of O=90, MN = 9 feet, and OM = 4.2 feet. Find the measure of N to the nearest degree. Marquis wrote the linear regression equation y=1.245x-3.685 to predict the cost, y, of x songs purchased. Which is the best estimate of number of songs that marquis purchased? Can someone help and explain A and B please... Please Help!!! Explain how to solve!!! Extra points and brainiest will be given Find the constant of proportionality (r)(r)left parenthesis, r, right parenthesis in the equation y=rxy=rxy, equals, r, x.The quantities xxx and yyy are proportional.xxxyyy777353535121212606060202020100100100Find the constant of proportionality (r)(r)left parenthesis, r, right parenthesis in the equation y=rxy=rxy, equals, r, x. The endocrine system sends hormones through the blood to control the activities of tissues and organs.Group of answer choicesa. Trueb. False How does the pressure of a container change as the volume is changed? Write a program whose input is a character and a string, and whose output indicates the number of times the character appears in the string.Ex: If the input is:n Mondaythe output is:1Ex: If the input is:z Today is Mondaythe output is:0Ex: If the input is:n It's a sunny daythe output is:2Case matters.Ex: If the input is:n Nobodythe output is:0n is different than N.This is what i have so far. #include #include using namespace std;int main() {char userInput;string userStr;int numCount;cin >> userInput;cin >> userStr;while (numCount == 0) {cout help me please ( ill give 20 points for this ) what is the reaction? A complex gear drawing done on a drawing sheet marked M-1 has many section views showing important interior details of the gear. One of the cutting-plane lines is marked at the ends with a callout in a circular bubble that says 7 above a line and M-3 below the line. To find this detail, you would What was the fate of both Spain and China in the 17th Which of the following is a good example of inflation?A. You get a flat tire on your car.B. You work three jobs in order to pay your bills.C.It cost you $50 to gas up your car this month. But last month it cost you$85.D.It cost you $85 to gas up your car this month. But last month it only costyou $50. Which of the following is generally NOT required for someone to become a policeofficer?Background checkClean recordGood physical conditionRecommendations from teachersem Cerdo 56 km caracol 48 m elefante 8 km araa 2 km conejo 48 km tortuga 156 m si todos los animales se desolazan al mismo punto se desplazan de izquierda a derecha que distancia recorrera cada uno despues del paso de una hora If your Cardiac Output is 10,000 mL per minute, and your heart rate is 120 bpm, whatis your stroke volume? I am thinking of a number. My number is between 20 and 30 My number and 12 have only one common factor. What number could I be thinking of? Give all three possible answers. what causes different colors to appear in the sky? Identify two Pythagorean triples using the known triple 9, 40 , 41. *Your answer